Linear Algebra - Projection of Vector

Click For Summary
The discussion revolves around finding the projection of vector u perpendicular to vector v. The user initially questions the answer key, believing it to be incorrect, and attempts to derive the perpendicular component using trigonometric relationships. They calculate the angle between the vectors and explore the relationship between the parallel and perpendicular components. Ultimately, the user acknowledges that the answer key is indeed correct, indicating a resolution to their confusion. The conversation highlights the importance of understanding vector projections in linear algebra.
YoshiMoshi
Messages
233
Reaction score
10

Homework Statement



I feel like this is a easy question but it seems the answer key doesn't seem to be right.

So say I have 2 vectors

equation 1.PNG


and I'm trying to find the projection of vector u perpendicular to the vector v

Homework Equations

The Attempt at a Solution



So I don't remember doing something like this before but I would assume that I start off by finding the projection of u in direction of v

equation 2.PNG

alright so this gives me
equation 3.PNG


So conceptually thinking I see that I have a vector u and a vector V|| sort of like this

IMG_20160408_153526779.jpg

So I can find the angle between u and V and I get 104.963 degrees

IMG_20160408_153856303.jpg


So to get the perpendicular component is it just

tan(theta) = (V perpendicular)/(V parallel)
so V perpendicular is
(V parallel)tan(theta) = V perpendicular?

IMG_20160408_154414914.jpg


I think is conceptually ok but I'm not sure because my answer key does V - V parallel which would appear to be wrong in my opinion because I don't see how that would give us V perpendicular, unless I'm not understanding what vector I'm looking for.

Thanks for any help.
 
Physics news on Phys.org
I see that apparently the answer key is correct and see why never mind
 
Question: A clock's minute hand has length 4 and its hour hand has length 3. What is the distance between the tips at the moment when it is increasing most rapidly?(Putnam Exam Question) Answer: Making assumption that both the hands moves at constant angular velocities, the answer is ## \sqrt{7} .## But don't you think this assumption is somewhat doubtful and wrong?

Similar threads

  • · Replies 1 ·
Replies
1
Views
2K
  • · Replies 1 ·
Replies
1
Views
4K
  • · Replies 4 ·
Replies
4
Views
2K
Replies
2
Views
2K
  • · Replies 5 ·
Replies
5
Views
2K
  • · Replies 7 ·
Replies
7
Views
1K
  • · Replies 2 ·
Replies
2
Views
2K
Replies
1
Views
1K
  • · Replies 12 ·
Replies
12
Views
2K
  • · Replies 24 ·
Replies
24
Views
3K